LSAT and Law School Admissions Forum

Get expert LSAT preparation and law school admissions advice from PowerScore Test Preparation.

 Administrator
PowerScore Staff
  • PowerScore Staff
  • Posts: 8916
  • Joined: Feb 02, 2011
|
#40158
Complete Question Explanation
(The complete setup for this game can be found here: lsat/viewtopic.php?t=8560)

The correct answer choice is (C)

This question asks you to determine what could be true if Y is at 1 PM. As with most Could Be True questions, the process of elimination is likely to be key: any answer choice that cannot be true will be incorrect.

If Y is at 1 PM, so is Z—in compliance with the second rule (in fact, Z must always be at 1 PM—that was our main inference):
PT73_Game_#2_#12_diagram 1.png
According to the first rule, M and L must be given in the same room, and M must be earlier than L. With the 1 PM slot occupied by Y and Z, L must given at 3 PM in one of the rooms. Since there is only one speech being given at 3 PM, we can establish a Dual Option for L and E in each of the two rooms:
PT73_Game_#2_#12_diagram 2.png
The remaining two variables—M and X—must therefore be given at 2 PM:
PT73_Game_#2_#12_diagram 3.png
The optimal approach here is similar to the one we took in Question #10. As with Question #10, the local setup does not take into account the implications of the last rule; nevertheless, it is sufficient to eliminate the four answer choices that cannot be true.

Answer choice (A) is incorrect, because only Z and Y can be given at 1 PM in either room.

Answer choice (B) is incorrect, because L must be given at 3 PM.

Answer choice (C) is the correct answer choice. While our local diagram clearly shows that M could be given at 2 PM, recall that we did not take into account the last rule of the game. One approach, therefore, would be to check if placing M at 2 PM in the Gold Room violates the conditional relationships established by the last rule. Since L must also be in the Gold Room with M, we need to ensure that Z and X are in the Rose Room in compliance with the last rule:
PT73_Game_#2_#12_diagram 4.png
This setup satisfies all rules, validating answer choice (C) as correct. A more efficient approach, however, would be to keep answer choice (C) as a contender, and proceed to examine answer choices (D) and (E). Proving that neither of them could be true is far easier than proving that answer choice (C) could be true.

Answer choice (D) is incorrect, because L is the only speech that can be given at 3 PM.

Answer choice (E) is incorrect, because only Z and Y can be given at 1 PM.

Get the most out of your LSAT Prep Plus subscription.

Analyze and track your performance with our Testing and Analytics Package.